Difference between revisions of "2014 AMC 12A Problems/Problem 14"

(Created page with "==Problem== Let <math>a<b<c</math> be three integers such that <math>a,b,c</math> is an arithmetic progression and <math>a,c,b</math> is a geometric progression. What is the sm...")
 
(added box+MAA notice)
Line 14: Line 14:
  
 
(Solution by AwesomeToad)
 
(Solution by AwesomeToad)
 +
 +
==See Also==
 +
{{AMC12 box|year=2014|ab=A|num-b=13|num-a=15}}
 +
{{MAA Notice}}

Revision as of 14:33, 8 February 2014

Problem

Let $a<b<c$ be three integers such that $a,b,c$ is an arithmetic progression and $a,c,b$ is a geometric progression. What is the smallest possible value of $c$?

$\textbf{(A) }-2\qquad \textbf{(B) }1\qquad \textbf{(C) }2\qquad \textbf{(D) }4\qquad \textbf{(E) }6\qquad$

Solution

We have $b-a=c-b$, so $a=2b-c$. Since $a,c,b$ is geometric, $c^2=ab=(2b-c)b \Rightarrow 2b^2-bc-c^2=(2b+c)(b-c)=0$. Since $a<b<c$, we can't have $b=c$ and thus $c=-2b$. then our arithmetic progression is $4b,b,-2b$. Since $4b < b < -2b$, $b < 0$. The smallest possible value of $c=-2b$ is $(-2)(-1)=2$, or $\boxed{\textbf{(C)}}$.

(Solution by AwesomeToad)

See Also

2014 AMC 12A (ProblemsAnswer KeyResources)
Preceded by
Problem 13
Followed by
Problem 15
1 2 3 4 5 6 7 8 9 10 11 12 13 14 15 16 17 18 19 20 21 22 23 24 25
All AMC 12 Problems and Solutions

The problems on this page are copyrighted by the Mathematical Association of America's American Mathematics Competitions. AMC logo.png